Your mother asks you to carry a load of wood to the fireplace and lower it to the hearth. In doing this you have used your arms as levers. Which statement describes this as a class three lever?





Question 13 options:

The wood in your hands was the fulcrum, the load your elbow, and the effort (input force) was supplied by your arm muscles.


The wood was the load in your hands, the fulcrum was your elbow and the effort (input force was supplied by your arm muscles.


The fulcrum was the muscles in your arms, the load was the wood in your hands and the effort (input force) was the elbow.

Answers

Answer 1

Answer: The correct answer is the weight of the wood.

Explanation:

Hope this helps


Related Questions

The normal force of a parked car on a level surface is 15,000 Newtons. What is the force of the car?

Answers

Answer:

The force of the car is 15000N.

Explanation:

The unit of force is Newtons (N), so based on the question, the force is 15000 Newtons.

Which electromagnetic wave has a lowest frequency?
Group of answer choices

A) x-rays

B) ultraviolet light

C) microwaves

D) infrared light

E) visible light

Answers

Answer:

E.visible lights

Explanation:

hope its attachments

A point charge of 5. 0 Ă— 10â€""7 C moves to the right at 2. 6 Ă— 105 m/s in a magnetic field that is directed into the screen and has a field strength of 1. 8 Ă— 10â€""2 T. What is the magnitude of the magnetic force acting on the charge? 0 N 2. 3 Ă— 10â€""3 N 23 N 2. 3 Ă— 1011 N.

Answers

The magnitude of the magnetic force acting on the charge which moves to the right is 0 N.

Given to us,

the charge [tex]q[/tex] =  [tex]5\times 10^{-7}[/tex] C,

the velocity [tex]v[/tex] = [tex]2.6\times 10^5[/tex]  m/sec,

the magnetic field [tex]B[/tex] = [tex]10^{-2}[/tex] T,

angle between the direction of v and B [tex]\theta[/tex] = 0,

Magnetic force is as important as the electrostatic or Coulomb force. The magnitude of the magnetic force F on a charge q moving at a velocity of v in a magnetic field of strength B is given by

[tex]\begin{aligned}F&=qvB\ sin\Theta\\&= 5\times10^{-7}\times2.6\times10^5\times10^{-2} \times sin(0)\\&= 0\ N\\\end{aligned}[/tex]

Hence, the magnitude of the magnetic force acting on the charge which moves to the right is 0 N

For More Information visit:

https://brainly.com/question/4111572

A single covalent bond is stronger than a single hydrogen bond so why does a group of polar molecules tend to have a higher boiling point than a group of non polar molecules

Answers

Answer:

this question makes no sense

Explanation:

like how do you get this question

Answer:They require more energy to break intermolecular forces hence polar molecules have higher melting points and boiling points than non-polar molecules of similar size, shape and number of electrons.

Explanation:

8 million electrons per second through an ohmic gas

Answers

the compare software

hope it's help

Answer:

you ask or answer?

Explanation:

thanks for point

The app claimed it has "Forbidden text"​

Answers

What??? I’m confused

if A and B are non zero vectors, is it possible for vector A×vector B and vector A.vector B both to be zero? Justify your answer​

Answers

Answer:

not able to understand the question

the purpose of many scientific investigations is to test a {n}

Answers

Hypothesis , I believe the answer is hypothesis

Answer:

Scientific investigation is a quest to find the answer to a question using the scientific method.

Explanation:

the scientific method is a systematic process that involves using measurable observations to formulate, test or modify a hypothesis.

What is the mass of an object that has a weight of 110N ?

Answers

Weight (W) = 110 NAcceleration due to gravity (g) = 9.8 m/s^2Let the mass of the object be m.By using the formula, W = mg, we get,110 N = 9.8 m/s^2 × mor, m = 110 N ÷ 9.8 m/s^2or, m = 11.2 Kg

Answer:

The mass of the object is 11.2 Kg.

Hope you could get an idea from here.

Doubt clarification - use comment section.

The mass of an object that has a weight of 110 N is 11.2kg.

What is Weight?

The weight of an object is defined as the force that acts on the object due to gravity. It is a vector quantity in some cases such as when the force of gravity acts on an object but it is also a scalar quantity where the force of gravity has a magnitude.

Weight is the gravitational force of attraction on an object due to the presence of another heavy object, such as the Earth or the Moon. It is expressed by multiplying the mass by the gravitational acceleration.

In above case,

Weight (W) = 110 N

Acceleration due to gravity (g) = 9.8 m/s^2

Let the mass of the object be m.

By using the formula,

W = mg,

[tex]110 N = 9.8 m/s^2 *m\\m = 110 N/ 9.8 m/s^2[/tex]

so, m = 11.2 Kg

Thus, the mass of an object that has a weight of 110 N is 11.2kg.

Learn more about Weight, here:

brainly.com/question/23312072

#SPJ2

how much force would be needed to push a box weighing 30 N up a ramp that ahas an ideal mechanical advantage of 3

Answers

Answer:

60n would be the needed force

The force needed to push a box weighing 30 N up a ramp that has an ideal mechanical advantage of 3 is equal to 10 N.

What is the mechanical advantage?

The mechanical advantage can be described as the ratio of the input force to the output force. The mechanical advantage of any machine can be determined by the ratio of the forces involved to do the work.

The ratio of the resistance force to the effort is called the actual mechanical advantage which will be comparatively less. The efficiency of a machine is always determined by equating the ratio of its output to its input.

The efficiency of the machine is equal to the ratio of the actual mechanical advantage (M.A.) and theoretical mechanical advantage. Mechanical advantage can be defined as the force produced by a machine to the force applied to it.

Given the load = 30 N and the ideal mechanical advantage = 3

Mechanical advantage = Load/ Effort

Input force or effort = Load/ M.A.

Force = 30/3

Input Force = 10 N

Learn more about the Mechanical advantage,  here:

brainly.com/question/16617083

#SPJ2

hi i need some help with this.....

Answers

Answer: D

Explanation: moogle helps lol.

the answer is C

Explanation:

In a plant cell, chloroplast makes sugar during the process of photosynthesis converting light energy into chemical energy stored in glucose. In mitochondria, through the process of cellular respiration breaks down sugar into energy that plant cells can use to live and grow.

how to solve for time given distance and velocity

Answers

Answer:

Well, I think you're talking about kinematics, especially uniform rectilinear motion. We know that there is a specific equation for that:

S = Vt + S0

With S being the distance, V the velocity, t the time and S0 the initial distance (initial displacement).

From this you can calculate t, if that's what you want.

A certain satellite is in circular orbit about the earth at an altitude of 550km. If the satellite makes a revolution in 110minutes, calculate its orbital speed ​

Answers

Answer:

Approximately [tex]6.59\times 10^{3}\; \rm m\cdot s^{-1}[/tex].

Explanation:

Look up the radius of the earth: approximately [tex]6.371 \times 10^{3}\; \rm km[/tex].

The radius of the orbit of this satellite is the of the radius of the earth (at ground level) plus the height of the satellite relative to ground level:

[tex]\begin{aligned}r &\approx 6.371 \times 10^{3}\; {\rm km} + 550\; {\rm km} \\ &= 6.921 \times 10^{3}\; \rm km\end{aligned}[/tex].

Convert the units of both distance and time to standard units:

Orbital radius:

[tex]\begin{aligned}r &\approx 6.921 \times 10^{3}\; {\rm km} \\ &= 6.921 \times 10^{3}\; {\rm km} \times \frac{10^{3}\; \rm m}{1\; \rm km} \\ &= 6.921 \times 10^{6}\; \rm m\end{aligned}[/tex].

Orbital period:

[tex]\begin{aligned}t &= 110\; \text{minute} \\ &= 110\; \text{minute} \times \frac{60\; \text{second}}{1\; \text{minute}} \\ &= 6.6 \times 10^{3}\; \text{second}\end{aligned}[/tex].

Calculate the circumference of this orbit:

[tex]\begin{aligned}& 2\, \pi\, r \\ \approx\; & 2 \, \pi \times 6.921 \times 10^{6}\; {\rm m} \\ \approx\; & 4.35 \times 10^{7}\; \rm m\end{aligned}[/tex].

Calculate the orbital speed (tangential) of this satellite:

[tex]\begin{aligned}v &= \frac{2\, \pi\, r}{t} \\ &\approx \frac{4.35 \times 10^{7}\; \rm m}{6.6 \times 10^{3}\; \rm s} \\ &\approx 6.6 \times 10^{3}\; \rm m \cdot s^{-1}\end{aligned}[/tex].

Liquid X of volume 0.5m3 and density 900kgm-3 was mixed with liquid Y of volume 0.4m3 and density 800kgm-3. What was the density of the mixture?​

Answers

Answer:

Density of the mixture = 855.56kgm-3

Explanation:

Density = Mass / Volume

Volume of Liquid X = 0.5m³

Density of Liquid X = 900kgm-3

Mass of Liquid X = Density × Volume

= 900kgm-3 × 0.5m³ = 450kg

Volume of Liquid Y = 0.4m³

Density of Liquid Y = 800kgm-3

Mass of Liquid Y = Density × Volume

= 800kgm-3 × 0.4m³= 320kg

As X and Y are mixed, we add their masses and volumes together:

Mass = 770kg

Volume = 0.9m³

Now we can find the density of the mixture:

Density = 770kg / 0.9m³ = 855.56kgm-3 (rounded to the 2nd decimal)

A 2.0 kg block rests on a level surface. The coefficient of static friction is, and the coefficient of kinetic friction is A horizontal force, X, is applied to the block. As X is increased, the block begins moving. Describe how the force of friction varies as X increases from the moment the block is at rest to when it begins moving. Indicate how you could determine the force of friction at each value of X―before the block starts moving, at the point it starts moving, and after it is moving. Show your work.

ps. I had to change F to X because of brainly.

Answers

By Newton's second law, the net force acting on the block in the vertical direction is

∑ F [ver] = n - mg = 0

where n = magnitude of normal force and mg = weight of the block. It follows that n = mg.

When the block is at rest, the applied force X will not be enough to move the box until it can overcome the maximum mag. of static friction. If µ[s] is the coefficient of static friction, then the maximum mag. of the frictional force is

f = µ[s] n = µ[s] mg

The net horizontal force would be

∑ F [hor] = X - µ[s] mg = 0

so a minimum force of X = µ[s] mg is required to get the block moving. Any mag. smaller than this and the block stays at rest/in equilibrium.

Once the mag. of X exceeds µ[s] mg, the block will begin to move. At that point, if the coefficient of kinetic friction is µ[k], then the net force on the block is

∑ F [hor] = X - µ[k] mg = 0

so a minimum force of X = µ[k] mg would be needed to keep the block moving at constant speed, or otherwise X = µ[k] mg + ma if the block is accelerating with mag. a.

The principles here are captured in the attached plot.

What is the final temperature if it requires 5000 J of heat to warm 2.38892 x10-2 kg of water that starts at 5oC? Remember Cp for water is 4186 J/kgC

Answers

The final temperature of water is equal to 50.9999°C

Given the following data:

Mass = [tex]2.38892 \times 10^{-2}\;kg[/tex]Quantity of heat = 5000 J Specific heat capacity of water = 4186 J/kg°C

To determine the final temperature of water:

Mathematically, quantity of heat is given by the formula;

[tex]Q=mc\theta[/tex]

Where:

Q represents the quantity of heat.m represents the mass of an object.c represents the specific heat capacity.∅ represents the change in temperature.

Substituting the given parameters into the formula, we have;

[tex]5000=2.38892 \times 10^{-2}\times 4186 \times \theta\\\\5000=100.0001912 \theta\\\\ \theta=\frac{5000}{100.0001912} \\\\ \theta=49.9999^{\circ}C[/tex]

For the final temperature:

[tex]\theta = T_2 - T_1\\\\T_2 = \theta+T_1\\\\T_2 = 49.9999 + 50[/tex]

Final temperature = 50.9999°C

Read more on final temperature here: https://brainly.com/question/2834175

2.
A swimmer swims 1 m/s due north against a current of 3 m/s due south. What is the resultant velocity of
the boat?

Answers

The resultant velocity of  the boat is equal to 2 m/s due south.

Given the following data:

Swimmer's speed = 1 m/s due northOcean current speed = 3 m/s due south.

To determine the resultant velocity of  the boat:

Resultant velocity can be defined as the sum (addition) of each of the vector velocity acting on a physical or an object. Thus, it is simply a combination of two or more single vector velocity.

Note: When the velocities are acting in the same direction, you will add them up while you will subtract when the velocities are acting in opposite directions.

In this scenario, the velocities are acting in opposite directions. Thus, we would subtract as follows:

[tex]V = 3 - 1[/tex]

V = 2 m/s due south.

Read more on resultant velocity here: https://brainly.com/question/11182174

What are the three different social perspectives on sport

Answers

Answer:

functionalist theory,feminist theory. discipline of sociology

A ball falling from a height of 5 m was caught at some height after being reflected off the floor. Find the magnitude of the movement of the ball. If the distance traveled is equal to 7 meters.

Answers

Answer:

3m

Explanation:

I can't make a drawing right know, but it's all around vectors. First of all the ball travels down 5 meters, so we have a vector pointing down, then it is reflected and travels 2 meters up. You have to sum the vectors so you obtain that the distance traveled is: 5-2 meters = 3m

A 0.15 kg baseball moving at 20 m/s is stopped by a player in 0.010 s. What is the average force of the ball?

Answers

Answer: 300N

Explanation:

Impulse= Mass * Velocity

F.T = M * V

F= MV/T

F= (0.15*20)/ 0.01

F= 300N

A 50 kg driver of a car is traveling at 35 m/s when she hits a large deer. She strikes the air bag/seatbelt combination that brings her body to a stop in 1.1s. What is the impulse imparted to the driver of the car?


Answers

Answer:1590 N

Explanation:

Impulse =  change in momentum/time

change in momentum = 0 - 50 x 35 = -1750

-1750/1.1 = 1590 (3s.f.) N

Tim is pushing a heavy box across the floor. He is using 300N of force and can accelerate at 2m/s/s. What is the mass of the box?

-298 kg
-600 kg
-0.006 kg
-150 kg

Answers

F=300Na=2m/s^2

Using Newton's second law

[tex]\\ \sf\Rrightarrow F=ma[/tex]

[tex]\\ \sf\Rrightarrow 300=2m[/tex]

[tex]\\ \sf\Rrightarrow m=150kg[/tex]

Hey there!

The formula of “mass” in physics is:

m = F/a

Whereas “f” is ‘force’, “a” is ‘acceleration’, & “m” is your ‘mass’ of course.

mass = 300 Net force/2 acceleration

300 Net force/2 acceleration = m

mass = 150

Therefore, your answer is: 150 kg

Good luck on your assignment and enjoy your day!

~Amphitrite1040:)


[tex] \huge{ \mathrm{question \hookleftarrow}}[/tex]


Calculate equivalent resistance of two resistors R1 and R2 in parallel where,

[tex] \sf R_ 1 = (6 \pm0.2 )\: \: ohms[/tex]
[tex] \sf R_ 2 = (3 \pm0.1 )\: \: ohms[/tex]

Answers

[tex]it \: was \: helpful \: to \: you[/tex]

# be careful#

How much energy has 4x 1010 m³ of water collected in a reservoir at a 2. 3. height of 100 m from the power house? What kind of energy is that? (Given, mass of 1 m³ of water = 1000 kg)​

Answers

Explanation:

[tex] \rule{999pt}{66646pt}[/tex]

4 What type of circuit is described by each of the following statements?
Answer series or parallel
a All components are connected end-to-end.
b. The current in the circuit divides so that some flows through one component
and the rest through another component
Two lamps are connected side by side so that each lights brightly
d The current has the same valur everywhere in the circuit
C

Answers

b. the current in the circuit divides so that some flows through one component

what voltage is measured across the 15 ohm resistor

Answers

Answer: The voltage across resistor is 75 Volts

A 15-ohm resistor has a 5-A current in it. What is the voltage across the resistor?





Mike drops a rock from the top of a suspension bridge. If the rock falls for 4.88 seconds,
how high is the bridge?

Answers

This question involves the concepts of the equations of motion in vertical direction.

The height of the bridge is "116.8 m".

We will use the second equation of motion in vertical direction to find out the height of the bridge.

[tex]h=v_it+\frac{1}{2}gt^2\\\\[/tex]

where,

h = hieght = ?

vi = initial speed = 0 m/s

t = time taken = 4.88 s

g = acceleration due to gravity = 9.81 m/s²

Therefore,

[tex]h=(0\ m/s)(4.88\ s)+\frac{1}{2}(9.81\ m/s^2)(4.88\ s)^2[/tex]

h = 116.8 m

Learn more about the equations of motion here:

https://brainly.com/question/5955789?referrer=searchResults

John throws a ball with a velocity of 30 m/s at an angle of 60 degrees. What is the horizontal component of the velocity?

a 30 m/s
b 0 m/s
c 25.9 m/s
d 15 m/s

Answers

The horizontal component of the velocity is equal to: D. 15 m/s.

Given the following data:

Velocity = 30 m/sAngle = 60°

To determine the horizontal component of the velocity:

The horizontal component of the velocity represents the influence of velocity  in displacing an object or projectile in the horizontal direction.

Mathematically, the horizontal component of velocity is given by the formula:

[tex]V_x = Vcos(\theta)[/tex]

Substituting the given parameters into the formula, we have;

[tex]\\\\V_x = 30cos(60)\\\\V_x = 30 \times 0.5[/tex]

Horizontal component, Vx = 15 m/s

Read more on horizontal component here: https://brainly.com/question/24681896

in what direction will the seesaw rotate and what will the sign of the angular acceleration be?

Answers

Answer:

It can rotate in any direction. The sign of the angular acceleration depends on how you set the reference system, it can be both negative or positive.

The gravitational force between two asteroids is 6.2 × 108 N. Asteroid Y has three times the mass of asteroid Z.

If the distance between the asteroids is 2100 kilometers, what is the mass of asteroid Y?

Answers

Answer:

1.1x10¹⁶ kg

Explanation:

Let m =  mass of asteroid y.

Because asteroid y has three times the mass of asteroid z, the mass of asteroid z is m/3.

Given:

F = 6.2x10⁸ N

d = 2100 km = 2.1x10⁶ m

Note that

G = 6.67408x10⁻¹¹ m³/(kg-s²)

The gravitational force between the asteroids is

F = (G*m*(m/3))/d² = (Gm²)/(3d²)

or

m² = (3Fd²)/G

    = [(3*(6.2x10⁸ N)*(2.1x10⁶ m)²]/(6.67408x10⁻¹¹ m³/(kg-s²))

   = 1.229x10³² kg²

m = 1.1086x10¹⁶ kg = 1.1x10¹⁶ kg (approx)

Other Questions
_____ puts a mother to sleep during childbirth to prevent her from experiencing the contractions, but she cannot help expel the baby and misses the delivery process. The formula for finding the perimeter of a rectangle is P = 2L + 2W. If a rectangle has a perimeter of 68 inches and the length is 14 inches longer than its width, find the width.Enter a number answer only. Solve the quadratic for x. Select all that apply.x? + 7x - 30 = 0O A. X=3O B. X=-6C. X=-10D. X=-3O E.X=5O F.x=6 Which of these events was the final factor in the end of the Ottoman Empire.O World WarO World War IIO exploration by SeaO invention of gunpowder The sum of three consecutive integers is 42. Which of the equations could represent this relationship? Select the two correct answers.x + (x + 1) + (x + 2) = 42x + (x + 2) + (x + 4) = 42 Question Is In The Picture 4. If a charged body is negatively charged, it has excess of _______ Please help me !!Nine more than eight times a number (x) is at least forty-nineWhats the inequality & the solution to the inequality is .. Use the correct order of operations to solve the problem below.26 - (5 + 4) - 20 4 The idea that Americans had a God-given right to expand westward was known as.. What is the perimeter of a square with a side length of 14 m? A. 46 m B. 56 m C. 70m D. 196 m Which sentence is punctuated correctly? (1 point)A "That dog is the Jones,' " my neighbor said.B "Those backpacks are my sisters," Callie said.C Please hand out the worksheets to the students'," the teacher told meD Put your science projects next to the seventh graders'," he instructed Add the polynomials: x2 + 2x + 1 and y2 + 2x + 3. Problem and Solution Sequence Comparison and Contrast Description Cause and Effect bre 1. Explains how something happens or is done, step by step. 2. Differences and similarities of two or more things are discussed. 3. A difficulty is described and an answer is offered. 4. An action and its results are discussed. 5. Describes how something looks or the arrangement of space. no What is the equation of the line in point - slope form with slope 3/4and passes through the point (5, -8)? HELPPPP!!!when will living seem worth it... 1. The teacher assigns homework after 7/10 of the lessonsA) unlikelyB) likelyC) certainD) equally likely Courtney has $21.50 to buy a gift for her brother. She found a stuffed animal that costs $19.65. With 10.1% sales tax added on, what is the sales tax? What is the total cost of the stuffed animal with tax included? Will Courtney have enough money to buy the gift?The sales tax is: $_________The total cost is: $_________Is it enough money? There are 3 cyan, 9 magenta and 15 yellow printer cartridges in a box.We pick one at random, then pick another one, without replacement.What is the probability that both of them will be cyan? The Falkland Islands are what location in relation to Mount Aconcagua?